Finding the limit for exponential function using Taylor Expansions

Click For Summary
The discussion focuses on finding the limit of the function (1 + 1/x^2)^x as x approaches infinity, with an emphasis on using Taylor expansions for the solution. The formal definition of the limit is provided, indicating that the goal is to show that the limit equals 1. Participants suggest using Taylor expansions to derive the polynomial and bound errors, but there is confusion regarding the exponent x. An alternative approach is proposed, recommending the use of logarithms and L'Hospital's rule for simplification. The conversation highlights the challenges of applying Taylor expansions in this context while exploring different methods to reach the limit.
georgetown13
Messages
7
Reaction score
0

Homework Statement


Determine the limit and then prove your claim.

limx\rightarrow\infty (1+\frac{1}{x^2} }) x

Homework Equations



I know that the formal definition that I need to use to prove the limit is:

{limx\rightarrow\infty (1+\frac{1}{x^2})x=1}={\forall \epsilon>0, \exists N > 0, \ni x>N \Rightarrow |f(x)-1|< \epsilon}

The Attempt at a Solution



We have to use Taylor Expansions to find the Taylor polynomial of f(x) and bound the errors to solve for \delta, given \epsilon >0.
The "x" exponent, however, is throwing me off. Could someone help guide me through the Taylor expansion of f(x)? I'd greatly appreciate it!
 
Physics news on Phys.org
georgetown13 said:

Homework Statement


Determine the limit and then prove your claim.

limx\rightarrow\infty (1+\frac{1}{x^2} }) x


Homework Equations



I know that the formal definition that I need to use to prove the limit is:

{limx\rightarrow\infty (1+\frac{1}{x^2})x=1}={\forall \epsilon>0, \exists N > 0, \ni x>N \Rightarrow |f(x)-1|< \epsilon}

The Attempt at a Solution



We have to use Taylor Expansions to find the Taylor polynomial of f(x) and bound the errors to solve for \delta, given \epsilon >0.
The "x" exponent, however, is throwing me off. Could someone help guide me through the Taylor expansion of f(x)? I'd greatly appreciate it!

Not offering to try the Taylor thing here; are you required to do it that way? Otherwise, I would let

y = \left( 1 + \frac 1 {x^2}\right)^x

and work with \ln(y) using L'Hospital's rule.
 
Question: A clock's minute hand has length 4 and its hour hand has length 3. What is the distance between the tips at the moment when it is increasing most rapidly?(Putnam Exam Question) Answer: Making assumption that both the hands moves at constant angular velocities, the answer is ## \sqrt{7} .## But don't you think this assumption is somewhat doubtful and wrong?

Similar threads

  • · Replies 13 ·
Replies
13
Views
4K
Replies
2
Views
2K
Replies
4
Views
2K
Replies
2
Views
2K
Replies
17
Views
2K
  • · Replies 1 ·
Replies
1
Views
2K
  • · Replies 10 ·
Replies
10
Views
2K
Replies
10
Views
2K
  • · Replies 7 ·
Replies
7
Views
2K
Replies
16
Views
2K